Diễn Đàn MathScopeDiễn Đàn MathScope
  Diễn Đàn MathScope
Ghi Danh Hỏi/Ðáp Community Lịch

Go Back   Diễn Đàn MathScope > Sơ Cấp > Tài Liệu > Đề Thi > Đề Thi HSG Cấp Tỉnh ở Việt Nam

News & Announcements

Ngoài một số quy định đã được nêu trong phần Quy định của Ghi Danh , mọi người tranh thủ bỏ ra 5 phút để đọc thêm một số Quy định sau để khỏi bị treo nick ở MathScope nhé !

* Nội quy MathScope.Org

* Một số quy định chung !

* Quy định về việc viết bài trong diễn đàn MathScope

* Nếu bạn muốn gia nhập đội ngũ BQT thì vui lòng tham gia tại đây

* Những câu hỏi thường gặp

* Về việc viết bài trong Box Đại học và Sau đại học


Trả lời Gởi Ðề Tài Mới
 
Ðiều Chỉnh Xếp Bài
Old 13-10-2018, 08:30 AM   #1
sieunhanbachtang
+Thành Viên+
 
Tham gia ngày: Oct 2018
Bài gởi: 28
Thanks: 14
Thanked 2 Times in 2 Posts
Bài 8 phần số học:Ta có thể tổng quát hóa kết quả bài toán như sau:
Cho 2 đa thức $P(x),Q(x) \in Z[x]$ thỏa mãn $P,Q$ nguyên tố cùng nhau.Đặt $a_n=(|P(n)|,|Q(n)|) \forall n\in Z^+$,khi đó dãy $a_n$ tuần hoàn
CM: Do $P,Q$ ntcn,nên tồn tại $R,S\in Z[x]$ sao cho $PR+QS=c$
Do đó $a_n|c \forall n$
Mặt khác $P(n+c)=P(n)(mod c),Q(n+c)=Q(n)(mod c)$,tồn tại $a,b$ để $aP(n)+bQ(n)=a_n=aP(n+c)+b(Q(n+c)(mod c)$ nên $a_n=aP(n+c)+bP(n+c)(mod a_{n+c})$ hay $a_{n+c}|a_n$hoàn toàn tương tự ta cx có $a_n|a_{n+c}$ nên dãy $a_n$ tuần hoàn chu kì $c$
[RIGHT][I][B]Nguồn: MathScope.ORG[/B][/I][/RIGHT]
 

thay đổi nội dung bởi: sieunhanbachtang, 13-10-2018 lúc 08:33 AM
sieunhanbachtang is offline   Trả Lời Với Trích Dẫn
Old 19-09-2018, 01:30 AM   #2
nguyenhaan2209
+Thành Viên+
 
Tham gia ngày: Jul 2017
Bài gởi: 7
Thanks: 1
Thanked 6 Times in 4 Posts
Đề SP & KHTN
Số học
$1$.Câu $a$ đơn thuần là chọn số câu $b$ giả sử phản chứng $m=p.S$ thì chú ý $p,S<=m/3$ nên chọn đc $xi-xj=p$ và $xi+xj-1=2S$ từ đó có ĐPCM
$2$. Đặt $a1=m$ chia khoảng và chú ý $a_x=y$ và $a_y=x$ nên $2m-2|n$
$3$. Phản chứng là vô hạn. Do tích $= 2018!$, tổng $2018$ số $<=2018!+2017$, mà dễ thấy qua mỗi bước tổng đều tăng nên dễ thấy vô lí
$4$.Khai triển $p^{2^{n+1}}-1/p-1$ ta được $(p+1)...(p^{2^n}+1)$ mà CM đc các thừa số trên nguyên tố cùng nhau nên $p+1=a^2$, $p^2+1=b^2$ nên $a^4-2a^2+2=b^2$ dùng pp chặn thấy chỉ có $a=1$ là t/m
$5$. Đặt $u_n=a_n+1$ thì $u_1=1, u_2=4$ và $u_{n+2}=7u_{n+1}-u_{n}-2$ bằng quy nạp đơn giản thu được $u_n=F_{2n-1}^2$ nên $a_n+5=F_{2n-1}^2+4$ chú ý rằng $2027$ là số nguyên tố có dạng $4k+3$ nên theo bổ đề quen thuộc ta có ĐPCM
Đại số
$1$. Dùng $ab(a+b)<=a^3+b^3$ và CT tính $1^3+...+n^3$
$2$. Sử dụng phản chứng thì đc $f$ có $2$ hoặc $4$ nghiệm cân bằng hệ số đc ĐPCM
$5$. Xuất phát từ bài toán quen thuộc $P(x)^2=P(x^2)$: Xét bậc $2n, 2n-1$ của VP thì suy ra $P(x)=x^k$ thay vào được $C=0$ đáp số là $0,1,x^n$
$6$. Đặt $x=a/b+c$ thì ta tính được $P<=2-6abc/(a+b)(b+c)(c+a)<=5/4$ và $P>=0$ do $a,b,c$ không âm
[RIGHT][I][B]Nguồn: MathScope.ORG[/B][/I][/RIGHT]
 

thay đổi nội dung bởi: nguyenhaan2209, 19-09-2018 lúc 07:17 AM
nguyenhaan2209 is offline   Trả Lời Với Trích Dẫn
The Following 2 Users Say Thank You to nguyenhaan2209 For This Useful Post:
kimlinh (19-09-2018), ncthanh (19-09-2018)
Old 19-09-2018, 03:45 PM   #3
ncthanh
Moderator
 
Tham gia ngày: Oct 2017
Đến từ: THPT Chuyên Bảo Lộc
Bài gởi: 17
Thanks: 51
Thanked 10 Times in 7 Posts
Trích:
Nguyên văn bởi MATHSCOPE View Post

$\boxed{7}$ [Ninh Bình] Cho đa thức $P(x)$ có hệ số nguyên và $a,b,c$ là các số nguyên thỏa mãn $P(a)=1,P(b)=2,P(c)=3$. Chứng minh rằng: $a+c=2b$.
Ta có tính chất quen thuộc sau: " Với $P(x)$ là đa thức hệ số nguyên và $x$, $y$ là các số nguyên thì $x-y$ là ước của $P(x)-P(y)$ "
Áp dụng tính chất trên ta suy ra $b-a\mid P(b)-P(a)=1$ và $c-b\mid P(c)-P(b)=1$, nên $b-a$ và $c-b$ nhận giá trị bằng $1$ hoặc $-1$, nhưng $P(a)\ne P(c)$ nên $a\ne c$, tức là $(b-a)+(c-b)\ne 0$, nên $b-a$ và $c-b$ cùng bằng $1$ hoặc cùng bằng $-1$, do đó $a+c=2b$, ta có điều cần chứng minh.
[RIGHT][I][B]Nguồn: MathScope.ORG[/B][/I][/RIGHT]
 
ncthanh is offline   Trả Lời Với Trích Dẫn
The Following User Says Thank You to ncthanh For This Useful Post:
MATHSCOPE (20-09-2018)
Old 19-09-2018, 11:47 PM   #4
vnt.hnue
Moderator
 
Tham gia ngày: Sep 2016
Bài gởi: 23
Thanks: 26
Thanked 15 Times in 8 Posts
$\boxed{10}$ [Hải Phòng] Giải phương trình sau với 2018 dấu phân số\[1 + \dfrac{1}{{1 + \dfrac{1}{\begin{array}{l}
1 + \\
\quad\ddots \;1 + \dfrac{1}{x}\\

\end{array}}}} = x.\]
- Nhận xét 1 : Nếu $a$ là nghiệm của phương trình $f(x)=x$ thì cũng là nghiệm của $f(f(x))=x$ (vì $f(f(a))=f(a)=a$ . Áp dụng nhận xét liên tiếp 2017 lần với $f(x)=1+\frac{1}{x}$, ta suy ra 2 nghiệm của phương trình $1+\frac{1}{x}=x$ cũng là nghiệm của phương trình đề bài.
- Nhận xét 2: Phương trình đã cho thực chất là phương trình bậc 2 nên có tối đa 2 nghiệm. Từ đó ta kết luận được phương trình có 2 nghiệm là $\frac{1-\sqrt{5}}{2}$; $\frac{1+\sqrt{5}}{2}$
[RIGHT][I][B]Nguồn: MathScope.ORG[/B][/I][/RIGHT]
 

thay đổi nội dung bởi: vnt.hnue, 20-09-2018 lúc 04:23 PM
vnt.hnue is offline   Trả Lời Với Trích Dẫn
Old 20-09-2018, 04:32 PM   #5
nkhanh75
+Thành Viên+
 
Tham gia ngày: Jun 2013
Bài gởi: 2
Thanks: 0
Thanked 0 Times in 0 Posts
Trích:
Nguyên văn bởi vnt.hnue View Post
$\boxed{10}$ [Hải Phòng] Giải phương trình sau với 2018 dấu phân số\[1 + \dfrac{1}{{1 + \dfrac{1}{\begin{array}{l}
1 + \\
\quad\ddots \;1 + \dfrac{1}{x}\\

\end{array}}}} = x.\]
........
- Nhận xét 2: Phương trình đã cho thực chất là phương trình bậc 2 nên có tối đa 2 nghiệm.
Cái nhận xét 2 này chưa ổn chút nào
[RIGHT][I][B]Nguồn: MathScope.ORG[/B][/I][/RIGHT]
 
nkhanh75 is offline   Trả Lời Với Trích Dẫn
Old 20-09-2018, 12:34 AM   #6
vnt.hnue
Moderator
 
Tham gia ngày: Sep 2016
Bài gởi: 23
Thanks: 26
Thanked 15 Times in 8 Posts
$\boxed{3}$ [Lạng Sơn] Cho $a,\,b,\,c$ là các số thực dương. Chứng minh rằng \[{\left( {\frac{a}{b} + \frac{b}{c} + \frac{c}{a}} \right)^2} \ge \left( {a + b + c} \right)\left( {\frac{1}{a} + \frac{1}{b} + \frac{1}{c}} \right).\]

Đặt $\frac{a}{b}=x,\frac{b}{c}=y,\frac{c}{a}=z$, có $xyz=1$. BĐT trở thành:
$(x+y+z)^{2}\ge x+y+z+\frac{1}{x}+\frac{1}{y}+\frac{1}{z}+3$
hay
$(x+y+z)^{2}\ge x+y+z+xy+yz+zx+3$
hay
$x^{2}+y^{2}+z^{2}+xy+yz+zx\ge x+y+z+3$
Sử dụng giả thiết $xyz=1$, bđt cần chứng minh có được trực tiếp từ $xy+yz+zx\ge 3$ và $x^{2}+y^{2}+z^{2}\ge x+y+z$
[RIGHT][I][B]Nguồn: MathScope.ORG[/B][/I][/RIGHT]
 
vnt.hnue is offline   Trả Lời Với Trích Dẫn
Old 20-09-2018, 03:35 PM   #7
ncthanh
Moderator
 
Tham gia ngày: Oct 2017
Đến từ: THPT Chuyên Bảo Lộc
Bài gởi: 17
Thanks: 51
Thanked 10 Times in 7 Posts
Trích:
Nguyên văn bởi MATHSCOPE View Post

$\boxed{9}$ [Sóc Trăng] Cho $x,\,y,\,z>0$ thỏa $x+y+z\le 1$, tìm giá trị nhỏ nhất của\[T = \frac{{\sqrt {{x^2}{y^2} + 1} }}{y} + \frac{{\sqrt {{y^2}{z^2} + 1} }}{z} + \frac{{\sqrt {{z^2}{z^2} + 1} }}{x}.\]
Ta có:
\[\begin{array}{l}
{\rm{ }}\frac{{\sqrt {{x^2}{y^2} + 1} }}{y} + \frac{{\sqrt {{y^2}{z^2} + 1} }}{z} + \frac{{\sqrt {{z^2}{x^2} + 1} }}{x}\\
= \sqrt {{x^2} + \frac{1}{{{y^2}}}} + \sqrt {{y^2} + \frac{1}{{{z^2}}}} + \sqrt {{z^2} + \frac{1}{{{x^2}}}} \\
= \sqrt {\left( {{x^2} + \frac{1}{{81{y^2}}}} \right) + \frac{{80}}{{81{y^2}}}} + \sqrt {\left( {{y^2} + \frac{1}{{81{z^2}}}} \right) + \frac{{80}}{{81{z^2}}}} + \sqrt {\left( {{z^2} + \frac{1}{{81{x^2}}}} \right) + \frac{{80}}{{81{x^2}}}} \\
\ge \sqrt {\frac{{2x}}{{9y}} + \frac{{80}}{{81}}{y^2}} + \sqrt {\frac{{2y}}{{9z}} + \frac{{80}}{{81}}{z^2}} + \sqrt {\frac{{2z}}{{9x}} + \frac{{80}}{{81}}{x^2}} {\rm{ }}\left( {AM - GM} \right)\\
\ge \sqrt {\frac{2}{9}{{\left( {\sqrt {\frac{x}{y}} + \sqrt {\frac{y}{z}} + \sqrt {\frac{z}{x}} } \right)}^2} + \frac{{80}}{{81}}{{\left( {\frac{1}{x} + \frac{1}{y} + \frac{1}{z}} \right)}^2}} \\
\ge \sqrt {\frac{2}{9}{{\left( {3\sqrt[3]{{\frac{x}{y}.\frac{y}{z}.\frac{z}{x}}}} \right)}^2} + \frac{{80}}{{81}}{{\left( {\frac{9}{{x + y + z}}} \right)}^2}} {\rm{ }}\left( {Minkowski} \right)\\
= \sqrt {82}
\end{array}\]
Vậy giá trị nhỏ nhất của $T$ là $\sqrt {82}$ đạt được khi $x = y = z = \frac{1}{3}$
[RIGHT][I][B]Nguồn: MathScope.ORG[/B][/I][/RIGHT]
 
ncthanh is offline   Trả Lời Với Trích Dẫn
Old 18-09-2018, 02:41 PM   #8
MATHSCOPE
Administrator

 
Tham gia ngày: Nov 2007
Bài gởi: 30
Thanks: 110
Thanked 183 Times in 68 Posts
Đề thi các trường chuyên và các tỉnh năm học 2018-2019-Lời giải và bình luận

Các bài toán Hình Học

$\boxed{1}$ [Chuyên ĐHSP Hà Nội] Cho tam giác $ABC$ nhọn, không cân, nội tiếp đường tròn $(O)$ $P,\,Q$ theo thứ tự là tâm đường tròn ngoại tiếp các tam giác $OAB,\,OAC$. $R$ là điểm đối xứng của $O$ qua $BC$. Gọi $X$ là giao điểm của $RP$ và $CP$, $Y$ là giao điểm của $RC$ và $BQ$. Chứng minh rằng $\widehat{BAX} = \widehat{YAC}$.

$\boxed{2}$ [Chuyên ĐHSP Hà Nội]Cho tam giác $ABC$ không cân nội tiếp đường tròn $O$, $I$ là tâm đường tròn nội tiếp. Gọi $E$ là giao điểm của $BI$ và $AC$, $F$ là giao điểm của $CI$ và $AB$; $M,\,N$ lần lượt là giao điểm thứ hai của $BI$ và $CI$ và đường tròn $O$. Đường thẳng $BI$ cắt đường tròn ngoại tiếp tam giác $BNF$ tại điểm thứ hai $P$, đường thẳng $CI$ cắt đường tròn ngoại tiếp tam giác $CME$ tại điểm thứ hai $Q$.
  1. Chứng minh rằng tứ giác $EFBQ$ nội tiếp một đường tròn.
  2. Qua $I$ kẻ đường thẳng $\Delta$ vuông góc với $BC$. Chứng minh rằng tâm đường tròn ngoại tiếp tứ giác $EFBQ$ nằm trên $\Delta$.

$\boxed{3}$ [Lạng Sơn] Cho hình chữ nhật $ABCD$, nội tiếp đường tròn $O$. Gọi $M,\,N$ lần lượt là trung điểm các cung nhỏ $BC,\,AD$. Gọi $I,\,J$ lần lượt là trung điểm $OM,\,ON$. Gọi $K$ là điểm dối xứng với $O$ qua $M$.
  1. Chứng minh răng tứ giác $BJDK$ nội tiếp đường tròn
  2. Gọi $P,\,Q$ lần lượt là hình chiếu vuông góc của $I$ lên $AB,\,AC$. Chứng minh rằng $AK\bot PQ$.

$\boxed{4}$ [Quảng Bình] Cho tam giác $ABC$ có $M$ là trung điểm $BC$. Gọi $D,\,E,\,F$ lần lượt là tiếp điểm của đường tròn ($I$) nội tiếp tam giác $ABC$ với các cạnh $AB,\,BC,\,AC$, đường thẳng $EF$ cắt đường thẳng $CI,\,BI,\,AM$ lần lượt tại $X,\,Y,\,N$. Chứng minuh rằng
  1. Giả sử $BC$ cố định và $A$ thay đổi trong mặt phẳng sao cho $\widehat{BAC}=\alpha,\;\ 0 < \alpha< 180^o$. Chứng minh độ dài đoạn $XY$ không đổi.
  2. Giả sử tam giác $ABC$ không cân, chứng minh rằng ba điểm $N,\,I,\,D$ thẳng hàng và $\dfrac{{NX}}{{NY}} = \dfrac{{AC}}{{AB}}$.

$\boxed{5}$ [Quảng Bình] Cho tam giác $ABC$ nhọn không cân, ($AB<AC$) có $H$ là trực tâm, nội tiếp đường tròn $(O)$ $BE,\,CF$ là các đường cao của tam giác $ABC$ $(E\in AC,\,F\in AB )$. Đường thẳng $EF$ cắt $BC$ tại $G$, đường thẳng $AG$ cắt đường tròn $(O)$ tại $M$.
  1. Gọi $T$ trung điểm $BC$, chứng minh rằng $GH\bot AT$.

  2. Lấy điểm $P$ nào đó trên tia $BC$ ($P$ nằm ngoài đoạn $PC$). Đường tròn $(O)$ cắt $AP$ tại $I$ và cắt đường tròn đường kính $AP$ tại $Q$ ($I,\,Q$ đều khác $A$) $AQ$ cắt $BC$ tại $J$. Chứng minh rằng đường thẳng $IJ$ luôn đi qua một điểm cố định.

$\boxed{6}$ [Sài Gòn] Cho $AB$ là một dây cố định khác đường kính của đường tròn $(O)$ cố định. Gọi $M$ là trung điểm của cung nhỏ $AB$. Xét đường tròn $\left(O' \right)$ thay đổi tiếp xúc $(O)$ tại một điểm thuộc cung lớn $AB$ ($\left(O' \right)$ khác phía đối với $M$ so với đường thẳng $AB$). Các đường thẳng qua $M$ vuông góc với $O'A$ và $O'B$ cắt $AB$ tại các điểm $C,\,D$.
  1. Chứng minh rằng $AB=2CD$.

  2. Gọi $T$ là một điểm thuộc $O'$ sao cho góc $ATB=90^o$. Giả sử tiếp tuyến của $\left(O' \right)$ tại $T$ cắt đoạn $AB$ tại $N$ và đường thẳng $MN$ cắt $\left(O\right)$ tại $K$ khác $M$. Vẽ đường tròn $M,\,K$ tiếp xúc ngoài với $\left(O' \right)$ tại $S$. Chứng minh rằng $S$ luôn di chuyển trên một đường tròn cố định khi $\left(O' \right)$ thay đổi.

$\boxed{7}$ [Sài Gòn] Cho tam giác $ABC$ nhọn, không cân và nội tiếp $(O)$. Một đường tròn $(J)$ thay đổi đi qua $B,\,C$ và cắt các đoạn $AB,\,AC$ lần lượt tại $D,\,E$. Trên đường thẳng $BC$ lấy hai điểm phân biệt $R,\,S$ sao cho $(DER)$ và $(DES)$ tiếp xúc với đường thẳng $BC$. giả sử $(ADE)$ cắt $(O)$ tại $M$ khác $A$. Gọi $(O')$ là đường tròn ngoại tiếp tam giác $RSM$.
  1. Chứng minh rằng đường tròn $(O')$ là đường tròn ngoại tiếp tam giác $RSM$.

  2. Chứng minh rằng điểm $O'$ luôn di động trên một đường thẳng cố định khi $(J)$ thay đổi.

$\boxed{8}$ [Hà Nội] Cho hai đường tròn $(O)$ và $\left( {O'} \right)$ cắt nhau tại $A,\,B$. Qua $A$ kẻ hai đường thẳng $A_1$ và $A_2$, đường thẳng $A_1$ cắt hai đường tròn $(O)$ và $\left( {O'} \right)$ lần lượt tại $C$ và $D$;đường thẳng $A_2$ cắt hai đường tròn $(O)$ và $\left( {O'} \right)$ lần lượt tại $E$ và $F$($C,\,D,\,E,\,F$ khác $A$). Các đường trung trực $CD$ và $EF$ cắt nhau tại $K$. Đường thẳng $d$ thay đổi đi qua $K$ cắt đường tròn $\left( {O'} \right)$ tại $P,\,Q$. Chứng minh rằng trực tâm tam giác $APQ$ luôn nằm trên một đường tròn cố định.

Sẽ update thường xuyên..
[RIGHT][I][B]Nguồn: MathScope.ORG[/B][/I][/RIGHT]
 
MATHSCOPE is offline   Trả Lời Với Trích Dẫn
The Following 2 Users Say Thank You to MATHSCOPE For This Useful Post:
huynhcongbang (03-10-2018), ncthanh (21-09-2018)
Old 21-09-2018, 02:53 PM   #9
ncthanh
Moderator
 
Tham gia ngày: Oct 2017
Đến từ: THPT Chuyên Bảo Lộc
Bài gởi: 17
Thanks: 51
Thanked 10 Times in 7 Posts
Trích:
Nguyên văn bởi MATHSCOPE View Post

$\boxed{2}$ [Chuyên ĐHSP Hà Nội]Cho tam giác $ABC$ không cân nội tiếp đường tròn $O$, $I$ là tâm đường tròn nội tiếp. Gọi $E$ là giao điểm của $BI$ và $AC$, $F$ là giao điểm của $CI$ và $AB$; $M,\,N$ lần lượt là giao điểm thứ hai của $BI$ và $CI$ và đường tròn $O$. Đường thẳng $BI$ cắt đường tròn ngoại tiếp tam giác $BNF$ tại điểm thứ hai $P$, đường thẳng $CI$ cắt đường tròn ngoại tiếp tam giác $CME$ tại điểm thứ hai $Q$.
  1. Chứng minh rằng tứ giác $EFBQ$ nội tiếp một đường tròn.
  2. Qua $I$ kẻ đường thẳng $\Delta$ vuông góc với $BC$. Chứng minh rằng tâm đường tròn ngoại tiếp tứ giác $EFBQ$ nằm trên $\Delta$.
1. Ta có: $\frac{{IF}}{{IP}} = \frac{{IB}}{{IN}} = \frac{{IC}}{{IM}} = \frac{{IE}}{{IQ}}$, suy ra $IF.IQ = IE.IP$, nên tứ giác $EFPQ$ nội tiếp.

2. Trước tiên ta phát biểu bổ đề sau: " Cho tam giác $ABC$ nhọn nội tiếp đường tròn $(O)$, ngoại tiếp đường tròn (I); đường phân giác của góc $B$ cắt $AC$ tại $E$ và cắt đường tròn (O) lần nữa tại $K$; đường phân giác góc $C$ cắt $AB$ tại $F$ và cắt đường tròn (O) lần nữa tại $L$. Khi đó ta có $IE.BK=IF.CL$"



Chứng minh: Kí hiệu $a$, $b$, $c$ lần lượt là độ dài các cạnh $BC$, $CA$, $AB$; $p$ là nửa chu vi tam giác $ABC$ và $r$ là bán kính đường tròn nội tiếp tam giác $ABC$. Ta có $BK.BE=ca$, đồng thời $\frac{{IE}}{{BE}} = \frac{{\left[ {AIC} \right]}}{{\left[ {ABC} \right]}} = \frac{{\frac{1}{2}br}}{{pr}} = \frac{b}{{a + b + c}}$, nên $BK.IE = \frac{{abc}}{{a + b + c}}$, tương tự $CL.IF = \frac{{abc}}{{a + b + c}}$, suy ra $IE.BK=IF.CL$. Hoàn tất chứng minh.

Trở lại bài toán.



Gọi $K$ là tâm đường tròn ngoại tiếp tứ giác $EFPQ$ . Ta có $K$ nằm trên $\Delta $ nếu và chỉ nếu $IK$ vuông góc với $BC$, hay:\[I{B^2} - I{C^2} = K{B^2} - K{C^2} \qquad(1).\]
Điều này tương đương với: \[ IB.BP + IB.IP - IC.CQ - IC.IQ = BP.BE - CQ.CF.\]
Hay: \[\begin{array}{l}
{\rm{ }}IB.IP - IC.IQ = BP.IE - CQ.IF\\
\Leftrightarrow IN.IF - IE.IM = BP.IE - CQ.IF\\
\Leftrightarrow IE.\left( {BP + IM} \right) = IF.\left( {CQ + IN} \right)\\
\Leftrightarrow IE.\left( {BM - PI} \right) = IF.\left( {CN - IQ} \right)\\
\Leftrightarrow IE.BM = IF.CN. \qquad(2)
\end{array}\]
Tới đây, áp dụng bổ đề vừa chứng minh ta có ngay đẳng thức $(2)$ đúng, tức là đẳng thức $(1)$ đúng, do đó $K$ thuộc $\Delta$.
[RIGHT][I][B]Nguồn: MathScope.ORG[/B][/I][/RIGHT]
 
Hình Kèm Theo
Kiểu File : png bổ đề.PNG (34.6 KB, 124 lần tải)
Kiểu File : png bài làm.PNG (30.6 KB, 123 lần tải)
ncthanh is offline   Trả Lời Với Trích Dẫn
The Following 2 Users Say Thank You to ncthanh For This Useful Post:
Le khanhsy (25-09-2018), nguyenhaan2209 (21-09-2018)
Old 21-09-2018, 10:09 AM   #10
Le khanhsy
Super Moderator
 
Tham gia ngày: Oct 2017
Bài gởi: 48
Thanks: 52
Thanked 57 Times in 30 Posts
Trích:
Nguyên văn bởi MATHSCOPE View Post
$\boxed{8}$ [Ninh Bình] Cho ba số thực dương $a,\,b,\,c$. Chứng minh bất đẳng thức\[\left( {a + b + c} \right)\left( {\frac{1}{a} + \frac{1}{b} + \frac{1}{c}} \right) + 4\sqrt 2 \left( {\frac{{ab + bc + ca}}{{{a^2} + {b^2} + {c^2}}}} \right) \ge 9 + 4\sqrt 2 .\]
Xét một bất đẳng thức rất chặt sau đây
$$27(a-b)^2(b-c)^2(c-a)^2=4(p^2-3q)^3-(2p^3-9pq+27r)^2 \ge 0$$
Chúng ta thu được
\[- 2(p^2-3q)\sqrt{p^2-3q} \le 2p^3-9pq+27r \le 2(p^2-3q)\sqrt{p^2-3q},\]
từ đây ta có
\[\dfrac{-2p^3+9pq-2(p^2-3q)\sqrt{p^2-3q}}{9} \le abc \le \dfrac{-2p^3+9pq+2(p^2-3q)\sqrt{p^2-3q}}{27}.\]
Bây giờ ta xét $a,b,c$ không âm, ta chuẩn hóa $a+b+c=3$ và khi đó tồn tại $t\in[0;1)$ sao cho $ab+bc+ca=3-3t^2$. Khi đó ta có
\[ (t+1)^2(1-2t)\le abc \le (t-1)^2(2t+1)\]
Vậy nên chúng ta cần chứng minh
\[\dfrac{3(3-3t^2)}{(t-1)^2(2t+1)} +\dfrac{4\sqrt{2}(3-3t^2)}{3+6t^2}\ge 9+4\sqrt{2}.\]
Việc nhóm các đối tượng cùng trọng số dễ dàng đưa chúng ta đến
\[\dfrac{24\sqrt{2}t^2\left(t+2-\dfrac{3}{\sqrt{2}} \right)^2}{(1-t)(2t+1)(2t^2+1)} \ge 0 .\]
Hoàn tất chứng minh. Đẳng thức xảy ra khi $a=b=c$ và hoán vị $\left(x,x,x\sqrt{2} \right).$

Ngoài ra chúng ta cũng có thể giải bằng thuần Cauchy-Schwarz một cách dễ dàng
[RIGHT][I][B]Nguồn: MathScope.ORG[/B][/I][/RIGHT]
 
Le khanhsy is offline   Trả Lời Với Trích Dẫn
The Following 2 Users Say Thank You to Le khanhsy For This Useful Post:
MATHSCOPE (21-09-2018), ncthanh (21-09-2018)
Old 21-09-2018, 12:06 PM   #11
nguyenhaan2209
+Thành Viên+
 
Tham gia ngày: Jul 2017
Bài gởi: 7
Thanks: 1
Thanked 6 Times in 4 Posts
Câu BĐT đề KHTN:
Với đề bài này ta nghĩ ngay tới phép thế $x=\frac{a}{b+c}, y=\frac{b}{c+a},z=\frac{c}{a+b}$
Biểu thức đã cho trở thành: $A=\sum \frac{ab}{(c+a)(c+b)}+\sqrt{\frac{abc}{(a+b)(b+c)( c+a)}}\sum \sqrt{\frac{a}{b+c}}$
Hay: $1+\sqrt{\frac{abc}{(a+b)(b+c)(c+a)}}(\sum \sqrt{\frac{a}{b+c}}-2\sqrt{\frac{abc}{(a+b)(b+c)(c+a)}})$
Đặt $\sqrt{\frac{abc}{(a+b)(b+c)(c+a)}}=t$
$1.$ Nếu a,b,c không có số nào bằng 0 thì $\sum \sqrt{\frac{a}{b+c}}\geq \sum \frac{a}{\sqrt{a(b+c)}}\geq 2>t$
Nếu có $1$ số $=0$, dễ thấy không đồng thời có $2$ số $=0$ nên WLOG gs $a=0$ thì
$\sum \sqrt{\frac{a}{b+c}}=\sum\sqrt{\frac{b}{c}}\geq 2>t$
Rõ ràng cả $2$ TH đều cho ta $A\geq1$ hay $minA=1$ dấu $"="$ xảy ra khi $(x,y,z)=(1,1,0)$ và các hoán vị
$2$. Ta có BĐT sau: $\sum \sqrt{\frac{2a}{b+c}}\geq 3$
$CM:$ Do vai trò đồng bậc, ta chuẩn hóa $a+b+c=3$
Bằng pp tiếp tuyến, ta CM đc $\sqrt{\frac{2a}{3-a}}\geq 1+\frac{a-1}{4}$
Biến đổi tương đương cái trên thành $(a-1)^2(a+5)\geq0$ hiển nhiên đúng
Áp dụng: $A\leq -2t^2+\frac{3}{\sqrt{2}}t+1$
Kháo sát hàm bậc $2$ trên $[0,\frac{1}{2\sqrt{2}}]$ có $A'=-4t+\frac{3}{\sqrt{2}}>0$ nên hàm đồng biến
Do vậy $A$ đạt cực đại tại $A(\frac{1}{2\sqrt{2}})=\frac{3}{2}$
Dấu $"="$ xảy ra khi $a=b=c=1$ hay $x=y=z=\frac{1}{2}$
------------------------------
Bài $12$ Đại số (Quảng Bình):
Gọi $x_1, x_2, . . . , x_n$ là các nghiệm thực của $P(x)$.
Nếu tồn tại $i ∈ {1, 2, . . . , n}$ sao cho $x_i<y$ thì $P(x_i)>0$ (mâu thuẫn vì $x_i$ là nghiệm của $P(x)$). Vậy ta có $0 < y ≤ x_i (i=1,n)$
Mặt khác ta có $P(x) = (x − x_1) (x − x_2). . .(x − x_n)$ và $n$ chẵn nên
$P(0) = (−1)^nx_1x_2 . . . x_n = x_1x_2 . . . x_n > 0$ (do n là số chẵn)
$P(y) = (y − x_1) (y − x_2). . .(y − x_n)= (x1 − y) (x2 − y). . .(xn − y) ≥ 0$
Điều cần chứng minh trở thành
$[y+\sqrt{(x_1 − y) (x_2 − y). . .(x_n − y)}]^n≤ x_1x_2 . . . x_n$
Áp dụng BĐT Holder ta được: $x_1x_2 . . . x_n = (y + x_1 − y) (y + x_2 − y). . .(y + x_n − y)≥[y+\sqrt{(x_1 − y) (x_2 − y). . .(x_n − y)}]^n$
Đây chính là điều phải chứng minh.
[RIGHT][I][B]Nguồn: MathScope.ORG[/B][/I][/RIGHT]
 

thay đổi nội dung bởi: nguyenhaan2209, 21-09-2018 lúc 12:26 PM Lý do: Tự động gộp bài
nguyenhaan2209 is offline   Trả Lời Với Trích Dẫn
The Following User Says Thank You to nguyenhaan2209 For This Useful Post:
ncthanh (21-09-2018)
Old 21-09-2018, 01:28 PM   #12
nguyenhaan2209
+Thành Viên+
 
Tham gia ngày: Jul 2017
Bài gởi: 7
Thanks: 1
Thanked 6 Times in 4 Posts
Câu $13$ Đại số (Quảng Bình)
$f(x-y)+f(xy)=f(x)-f(y)+f(x)f(y) \forall x\in R$ (1)
Gọi $P(u,v)$ là phép thế $x$ bởi $u$, $y$ bởi $v$ vào (1)
+ $P(x,0): f(0)(f(x)-2)=0$. TH $f(x)=2\forall x\in R$ ta thấy thỏa mãn do đó ta xét $f(0)=0$
+ $P(-1,-1): f(1)=f(-1)^2$ mà $P(0,1): f(-1)=-f(1)$ nên $f(1)=0$ hoặc $1$
$TH1: f(1)=0$ nên $f(-1)=0$
+ $P(x+1,1): f(x)=0$ thử lại ta thấy $f(x)=0 \forall x\in R$ thỏa mãn
TH$2$: $f(1)=1$
+ $P(x+1,1): f(x+1)=f(x)+1$ $(2)$
+ $P(x+1,y): f(x+1-y)+f(xy+y)=f(x+1)-f(y)+f(x+1)f(y)$
Từ $(2)\Rightarrow f(x-y)+1+f(xy+y)=f(x)+1-f(y)+f(x)f(y)+f(y)$
$\Rightarrow f(x-y)+f(xy+y)=f(x)+f(x)f(y)=f(x-y)+f(xy)+f(y)$
$\Rightarrow f(xy+y)=f(xy)+f(y) \Rightarrow f(x+y)=f(x)+f(y)$ $(3)$
+ $P(0,y):f(-y)=-f(y)$ do đó $f(x-y)=f(x)+f(-y)=f(x)-f(y)$
Thay vào $(1)$ thì $f(xy)=f(x)f(y)$ $(4)$
Từ $(3)(4)$ theo kết quả quen thuộc thì $f(x)=x$
Vậy bài toán có $3$ nghiệm hàm là $f(x)\equiv 0, f(x)\equiv 2, f(x)\equiv x \forall x\in R$
[RIGHT][I][B]Nguồn: MathScope.ORG[/B][/I][/RIGHT]
 
nguyenhaan2209 is offline   Trả Lời Với Trích Dẫn
The Following 2 Users Say Thank You to nguyenhaan2209 For This Useful Post:
MATHSCOPE (21-09-2018), ncthanh (21-09-2018)
Old 21-09-2018, 01:38 PM   #13
Le khanhsy
Super Moderator
 
Tham gia ngày: Oct 2017
Bài gởi: 48
Thanks: 52
Thanked 57 Times in 30 Posts
Trích:
Nguyên văn bởi MATHSCOPE View Post
$\boxed{1}$ [Chuyên ĐHSP Hà Nội] Cho $n$ là số nguyên lớn hơn $1$ và $\left\{x_1,\,x_2,\,\ldots,\,x_n\right\}$ là một hoán vị của $\left\{1,\,2,\,\ldots,\,n\right\}$,(tập hợp gồm $n$ số nguyên dương đầu tiên). Chứng minh rằng
\[\sum\limits_{k = 1}^n {k{x_k}\left( {k + {x_k}} \right)} \le \dfrac{{{n^2}{{\left( {n + 1} \right)}^2}}}{2}.\]
Trước tiên chúng ta có đẳng thức quen thuộc sau
\[1^3 + 2^3 + 3^3 + ... + n^3 = \dfrac{ n^2 (n+1)^2}{4} .\]
Theo giả thiết thì và kết hợp đẳng thức trên thì
\[ \sum_{k=1}^nx^3_k+ \sum_{k=1}^nk^3= \dfrac{ n^2 (n+1)^2}{2}.\]
Vì thế bài toán cần chứng minh
\[\sum\limits_{k = 1}^n {k{x_k}\left( {k + {x_k}} \right)} \le \sum_{k=1}^n \left( x^3_k+k^3\right),\]
hay
\[\sum_{k=1}^n \left( x_k-k\right)^2\left( x_k+k\right)\ge 0.\]
Bất đẳng thức trên là luôn đúng trên tập số tự nhiên.
[RIGHT][I][B]Nguồn: MathScope.ORG[/B][/I][/RIGHT]
 
Le khanhsy is offline   Trả Lời Với Trích Dẫn
The Following User Says Thank You to Le khanhsy For This Useful Post:
ncthanh (21-09-2018)
Old 21-09-2018, 04:11 PM   #14
nguyenhaan2209
+Thành Viên+
 
Tham gia ngày: Jul 2017
Bài gởi: 7
Thanks: 1
Thanked 6 Times in 4 Posts
Hình học
$1$. Chú ý tính chất điểm $Kosnita$ ($U$ là tâm $(OBC)$ thì $AU,BP,CQ$ đồng quy). Đoạn sau sử dụng hàng điểm điều hòa và chú ý qua phép đối xứng trục là pg góc $A$ bảo toàn tỉ số kép nên $AX,AY$ đẳng giác
$2$. Câu $a$ biến đổi góc đơn giản. Câu $b$ sử dụng định lí $4$ điểm ĐPCM tương đương với $SB^2-SC^2=IB^2-IC^2$ sau cùng cần cm $IE/IF=BN/CM$ điều này đúng dựa theo định lí $sin$ (lồng $IB, IC$ vào) và ta có ĐPCM
$3$.
$a) OJ.OK=ON.OM=OB.OD$ nên $BJDK$ nội tiếp
$b)$ $OI.OK=1/2R.2R=OA^2$ nên $OAI=OKA=KAB$ nên $AK$, $AI$ đẳng giác góc $BAC$ mà $AI$ là đường kính $(APQ)$ nên $AK$ là đg cao tức $AK$ vg $PQ$
$4$.
$a)$ Theo kết quả quen thuộc thì $BXYC$ nội tiếp mà $XY/BC=XY/2R=sinXBY=sinEFI=sinA/2$ luôn cố định do $A$ const
$b)$ Kẻ đgt qua $N//BC$ ta có $IEG$~$IFH$ do $EIG=END=HNF=HIF$ nên $IH/IG=IE/IF=1$ tức $N$ là tđ $HG$ suy ra $AN$ đi qua tđ $M$ của $BC$
$NX/NY=IX/IY.cosC/2:cosB/2=sinC/2cosC/2:sinB/2cosB/2=sinC/sinB=AC/AB$
$5.$
$a)$$DH.DA=DB.DC=DG.DT$ (Maclaurin) nên $H$ là trực tâm $ATG$ hay $GH$ vg $AT$
$b)$Gọi $IJ$ cắt $(O)$ tại $S$, $GS$ cắt $(O)$ tại $L$.
Ta có: $LC/LB=GC/GB.SB/SC$ (bổ đề quen thuộc) =$GC/GB.IC/IB.JB/JC$ (bổ đề cát tuyến)
$=EC/EA.FA/FB.IC/IB.JB/JC$ (định lí $Menelaus$)
$=(AC/AB)^2.DC/DB.sinPAC/sinPAB.JB/JD.JD/JC$ (định lí $sin$)
$=(AC/AB)^2.DC/DB.AB/AC.PC/PB.BD/PC.BP/DC$ ($JB/JD=JP/JC$) $=AC/AB$
Từ đó theo bổ đề cát tuyến ta có $AL$ đi qua tđ $T$ của $BC$, từ đó rõ ràng thấy $S$ cố định hay $IJ$ luôn đi qua điểm $S$ cố định
[RIGHT][I][B]Nguồn: MathScope.ORG[/B][/I][/RIGHT]
 

thay đổi nội dung bởi: nguyenhaan2209, 21-09-2018 lúc 04:14 PM
nguyenhaan2209 is offline   Trả Lời Với Trích Dẫn
The Following User Says Thank You to nguyenhaan2209 For This Useful Post:
ncthanh (21-09-2018)
Old 21-09-2018, 08:39 PM   #15
Nguyenhuyen_AG
+Thành Viên+
 
Nguyenhuyen_AG's Avatar
 
Tham gia ngày: Apr 2010
Bài gởi: 300
Thanks: 35
Thanked 307 Times in 151 Posts
Trích:
Nguyên văn bởi MATHSCOPE View Post
$\boxed{3}$ [Lạng Sơn] Cho $a,\,b,\,c$ là các số thực dương. Chứng minh rằng \[{\left( {\frac{a}{b} + \frac{b}{c} + \frac{c}{a}} \right)^2} \ge \left( {a + b + c} \right)\left( {\frac{1}{a} + \frac{1}{b} + \frac{1}{c}} \right).\]
Câu này là đề Iran 2005.

Giả sử $a = \max\{a,b,c\}.$ Ta xét hai trường hợp sau:

1. Nếu $b \geqslant c,$ áp dụng bất đẳng thức AM-GM
\[4(a+b+c)\left ( \frac{1}{a}+\frac{1}{b}+\frac{1}{c} \right ) \leqslant \left [\frac{a+b+c}{b}+b\left (\frac{1}{a}+\frac{1}{b}+\frac{1}{c} \right ) \right ]^2.\]
Ta quy bài toán về chứng minh
\[2\left ( \frac{a}{b}+\frac{b}{c}+\frac{c}{a} \right ) \geqslant \frac{a+b+c}{b}+b\left (\frac{1}{a}+\frac{1}{b}+\frac{1}{c} \right ),\]
thu gọn thành
\[\frac{(a-c)\left[b(b-c)+c(a-b)\right]}{abc} \geqslant 0.\]
Hiển nhiên đúng.

2. Nếu $b \leqslant c,$ đánh giá tương tự như trên
\[4(a+b+c)\left ( \frac{1}{a}+\frac{1}{b}+\frac{1}{c} \right ) \leqslant \left [\frac{a+b+c}{c}+c\left (\frac{1}{a}+\frac{1}{b}+\frac{1}{c} \right ) \right ]^2,\]
ta đưa bài toán về chứng minh
\[2\left ( \frac{a}{b}+\frac{b}{c}+\frac{c}{a} \right ) \geqslant \frac{a+b+c}{c}+c\left (\frac{1}{a}+\frac{1}{b}+\frac{1}{c} \right ),\]
tương đương với
\[\frac{(a-b)(2ca-ab-c^2)}{abc} \geqslant 0.\]
\[\frac{(a-b)\left[c(a-c)+a(c-b)\right]}{abc} \geqslant 0.\]
Hiển nhiên đúng.

Trong mọi trường hợp bất đẳng thức đều được chứng minh.

Ngoài ra thì
\[{\left( {\frac{a}{b} + \frac{b}{c} + \frac{c}{a}} \right)^2} \-\left( {a + b + c} \right)\left( {\frac{1}{a} + \frac{1}{b} + \frac{1}{c}} \right) = \sum \frac{(a+b)(a-b)^2}{ab^2} \geqslant 0.\]
------------------------------
Trích:
Nguyên văn bởi Le khanhsy View Post
Ngoài ra chúng ta cũng có thể giải bằng thuần Cauchy-Schwarz một cách dễ dàng
Bài này có thể giải thuần túy Cauchy-Schwarz bằng 2 cách.
[RIGHT][I][B]Nguồn: MathScope.ORG[/B][/I][/RIGHT]
 
__________________
Nguyen Van Huyen
Ho Chi Minh City University of Transport

thay đổi nội dung bởi: Nguyenhuyen_AG, 21-09-2018 lúc 09:17 PM Lý do: Tự động gộp bài
Nguyenhuyen_AG is offline   Trả Lời Với Trích Dẫn
The Following User Says Thank You to Nguyenhuyen_AG For This Useful Post:
ncthanh (21-09-2018)
Trả lời Gởi Ðề Tài Mới

Bookmarks


Quuyền Hạn Của Bạn
You may not post new threads
You may not post replies
You may not post attachments
You may not edit your posts

BB code is Mở
Smilies đang Mở
[IMG] đang Mở
HTML đang Tắt

Chuyển đến


Múi giờ GMT. Hiện tại là 01:45 AM.


Powered by: vBulletin Copyright ©2000-2024, Jelsoft Enterprises Ltd.
Inactive Reminders By mathscope.org
[page compression: 121.96 k/138.87 k (12.18%)]